-1
$\begingroup$

How to compute the expectation of $\frac{Y^L}{Y^L + (N-Y)^L}$ where $Y$ is Binomial(n,p)? If it is not exactly computable, then are their ways to approximate this qty?

$\endgroup$
4
  • 1
    $\begingroup$ For $L=2, N=n$, Mathematica gives an exact answer with the incomplete $\beta$ function: $-\frac{1}{2} i (1-p)^n \left(\frac{p}{p-1}\right)^{\left(\frac{1}{2}-\frac{i}{2}\right) n} \left((n-1) \left(\frac{p}{p-1}\right)^{i n} B_{\frac{p}{p-1}}\left(1-\left(\frac{1}{2}+\frac{i}{2}\right) n,n-1\right)-n \left(\frac{p}{p-1}\right)^{i n} B_{\frac{p}{p-1}}\left(1-\left(\frac{1}{2}+\frac{i}{2}\right) n,n\right)-(n-1) B_{\frac{p}{p-1}}\left(1-\left(\frac{1}{2}-\frac{i}{2}\right) n,n-1\right)+n B_{\frac{p}{p-1}}\left(1-\left(\frac{1}{2}-\frac{i}{2}\right) n,n\right)\right)$ $\endgroup$
    – user44143
    Apr 21, 2014 at 15:39
  • $\begingroup$ Mathematica also produces results for $N\neq n$ and larger values of $L$, albeit more complicated ones. $\endgroup$
    – Eckhard
    Apr 21, 2014 at 16:05
  • $\begingroup$ When Mathematica produces answers like this, it often seems to be a restatement of the problem, and not really progress. Does this expression help if you want to approximate the value? $\endgroup$ Apr 21, 2014 at 17:57
  • $\begingroup$ @DouglasZare, it does help for some purposes. The answer from Maple is more readable and understandable, but for large $n$ (and $L=2$), this will give you the answer more quickly and with better precision. $\endgroup$
    – user44143
    Apr 21, 2014 at 20:47

1 Answer 1

-5
$\begingroup$

It is exactly calculated in Maple by $$with(Statistics): Y := RandomVariable(Binomial(n, p)): $$ $$ M := Mean(Y^L/(Y^L+(N-Y)^L))\,assuming\, N::posint, L::posint$$ which produces $$\sum _{{\it \_t0}=0}^{n}{\frac {{{\it \_t0}}^{L}{n\choose {\it \_t0}}{ p}^{{\it \_t0}} \left( 1-p \right) ^{n-{\it \_t0}}}{{{\it \_t0}}^{L}+ \left( N-{\it \_t0} \right) ^{L}}} $$

$\endgroup$
2
  • 6
    $\begingroup$ This just looks like the definition of expectation. How does this help? $\endgroup$ Apr 21, 2014 at 21:18
  • $\begingroup$ @ Douglas Zare : So what? $\endgroup$
    – user64494
    Mar 17, 2015 at 5:05

Your Answer

By clicking “Post Your Answer”, you agree to our terms of service and acknowledge you have read our privacy policy.

Not the answer you're looking for? Browse other questions tagged or ask your own question.